Week 3 Quiz

Pataasin ang iyong marka sa homework at exams ngayon gamit ang Quizwiz!

Which two hemodynamic changes would increase the severity of aortic regurgitation? (select two) A. A decrease in heart rate B. An increase in heart rate C. An increase in the diastolic blood pressure D. A decrease in the diatolic blood pressure

A. A decrease in heart rate C. An increase in the diastolic blood pressure The decrease in heart rate increases the diastolic time which adversely affects the patient's condition by increasing the proportion of the stroke volume that regurgitates backward through the aortic valve. An increase in the diastolic blood pressure increases the backward pressure gradient which results in an increase in the proportion of stroke volume that regurgitates back into the left ventricle.

Which of the following would be appropriate in the anesthetic management of a patient with hypertrophic cardiomyopathy? A. Avoid an increase in myocardial contractility B. Maintain a slightly increased heart rate C. Restrict IV fluids to decrease the preload D. Administer nitrates to decrease the afterload

A. Avoid an increase in myocardial contractility Hypertrophic cardiomyopathy involves enlargement of the interventricular septum which results in left ventricular outflow obstruction. This obstruction is worsened by increased heart rate or increased myocardial contractility as well as decreases in preload or afterload. Anesthesia is usually maintained by controlled myocardial depression using volatile anesthetics.

The classic triad of symptoms associated with severe aortic stenosis include all of the following except A. midsystolic click on auscultation B. angina C. syncope D. congestive heart failure

A. midsystolic click on auscultation The classic triad of symptoms that accompanies severe aortic stenosis is angina, syncope, and congestive heart failure.

Which of the following interventions would be appropriate for a patient with mitral regurgitation, but NOT for a patient with mitral stenosis? A. Increase the heart rate B. Increase the afterload substantially C. Avoid an increase in pulmonary vascular resistance D. Increase preload slightly

A. Increase the heart rate In mitral regurgitation, your goal is to maintain an increased heart rate and decreased afterload while the goal in mitral stenosis is to maintain a normal or decreased heart rate and normal afterload. In both disorders, you should maintain normal sinus rhythm, avoid increases in pulmonary vascular resistance, and maintain preload at normal to increased levels.

Which disorder exhibits a higher incidence of mitral valve prolapse? A. Marfan syndrome B. COPD C. Myasthenia gravis D. Stevens-Johnson syndrome

A. Marfan syndrome MVP has a higher occurrence in patients with Marfan syndrome, systemic lupus erythematosus, rheumatic carditis, thyrotoxicosis, and myocarditis

Which alteration would be most likely to produce congestive heart failure or hypotension in the patient with aortic stenosis? A. New onset of atrial fibrillation B. Decrease in heartrate from 70 bpm to 50 bpm C. PVCs at a rate of 6-8 per minute D. New onset right bundle branch block

A. New onset of atrial fibrillation The decreased pressure gradient that exists between the left atrium and left ventricle in aortic stenosis limits left ventricular filling dramatically. Because left ventricular filling is so dependent upon atrial contraction, loss of atrial systole can result in congestive heart failure or hypotension.

Which of the following valvular lesions is most likely to result in eccentric left ventricular hypertrophy? A. Aortic stenosis B. Aortic regurgitation C. Mitral stenosis D. Tricuspid stenosis

B. Aortic regurgitation The hallmark characteristic of aortic regurgitation is eccentric left ventricular hypertrophy. Aortic stenosis is associated with the development of concentric left ventricular hypertrophy. Mitral stenosis and mitral regurgitation are both associated with overload of the left atrium with mitral stenosis often resulting in right ventricular hypertrophy as pressures are transmitted through the pulmonary vascular tree.

What hormone is released from the cardiac atria in response to increased atrial stretch and increases the renal excretion of sodium and water? A. Aldosterone B. Atrial natriuretic peptide C. Vasopressin D. Angiotensin

B. Atrial natriuretic peptide ANP is released from the cardiac atria due to increased atrial stretch. It exhibits vasodilatory effects and increases the renal excretion of sodium and water.

Which of the following statements regarding mitral valve stenosis is false? A. It is associated with a decreased left ventricular volume B. It is associated with a decreased left atrial pressure and increased right atrial pressure C. It can cause right ventricular failure D. It can result in pulmonary edema

B. It is associated with a decreased left atrial pressure and increased right atrial pressure As flow through the mitral valve opening into the left ventricle is decreased (thus decreasing the left ventricular volume), left atrial pressure increases, resulting in left atrial hypertrophy and distention. The increased pressure is transmitted into the pulmonary vasculature as the volume of pulmonary blood increases. This increased pulmonary vascular pressure represents an increase in right ventricular afterload and will cause right ventricular hypertrophy and failure. As pulmonary venous pressure increases above about 25 mmHg, fluid can leak into the pulmonary interstitial space resulting in a decrease in pulmonary compliance and increased work of breathing. If the change in pulmonary venous pressure occurs over a long period of time, an increase in pulmonary lymph flow can partially compensate for the fluid accumulation.

Which of the following symptoms of cardiac tamponade are indications of ventricular discordance? (select two) A. Beck's triad B. Kussmaul's sign C. Decreased voltage on the electrocardiogram D. Pulsus paradoxus

B. Kussmaul's sign D. Pulsus paradoxus Kussmaul's sign and pulsus paradoxus are both indicative of ventricular discordance (also known as ventricular dyssynchrony) that occurs due to the opposing response of the ventricles to filling during the respiratory cycle.

Which of the following decreases in mitral stenosis? A. Left atrial pressure B. Left ventricular volume C. Pulmonary artery pressure D. Aortic valve surface area

B. Left ventricular volume As the mitral valve orifice narrows, a pressure gradient develops across the valve as a compensatory mechanism to maintain flow through the valve. As the valve opening decreases and the pressure gradient increases, the flow of blood through the opening decreases and left ventricular volume decreases.

Which of the following statements does not correspond with an accurate understanding of the appropriate anesthetic management goals for the patient with aortic stenosis? A. Normal sinus rhythm should be maintained B. Myocardial depression reduces the severity of the lesion C. Hypotension should be treated aggressively D. Both bradycardia and tachycardia should be avoided

B. Myocardial depression reduces the severity of the lesion Because left ventricular filling is so dependent upon an appropriately-timed atrial contraction, maintenance of normal sinus rhythm is very important. The development of a junctional rhythm or atrial fibrillation can result in congestive heart failure and hypotension. Hypotension must be treated aggressively as the corresponding decrease in myocardial perfusion can decrease left ventricular function and result in a decrease in cardiac output which would further worsen hypotension. Sustained increases in heart rate must be avoided as the ventricles need adequate time for ventricular filling. Conversely, severe bradycardia can result in overfilling and ventricular distention.

The additional load that cardiac muscle faces after contraction begins is referred to as A. blood pressure B. afterload C. end-diastolic pressure D. cardiac stress

B. afterload The additional load that cardiac muscle faces after contraction begins is referred to as afterload.

A patient with mitral stenosis has a left atrial pressure of 27 mmHg. You would presume that the patient likely A. is asymptomatic B. has pulmonary hypertension C. has decreased preload D. would benefit from a balloon aortic valvotomy

B. has pulmonary HTN A mean left atrial pressure of 25 mmHg is required when the mitral area is less than 1 cm2. If this pressure is sustained, the patient will likely develop pulmonary hypertension.

Which of the following statements regarding aortic stenosis is true? A. Aortic stenosis is associated with an increase in left ventricular compliance B. Myocardial oxygen demand is not altered in aortic stenosis C. Aortic stenosis is associated with diastolic dysfunction D. Aortic stenosis does not alter arterial flow to the myocardium

C. Aortic stenosis is associated with diastolic dysfunction In aortic stenosis, left ventricular compliance decreases as the left ventricle hypertrophies, resulting in diastolic dysfunction. The myocardial demand is increased due to ventricular hypertrophy and the supply is decreased as the extraordinary compression of intramyocardial vessels during systole restricts arterial flow to the myocardium.

Which of the following symptoms are consistent with a diagnosis of cardiac tamponade? A. Hypertension B. Loud S2 heart sound over the apex C. Electrical alternans D. Bradycardia

C. Electrical alternans Beck's triad, the classic triplet of signs associated with cardiac tamponade, includes hypotension, distant heart sounds, and distended neck veins. Because of the obstruction of right atrial filling, central venous pressures increase resulting in distended neck veins unless severe hypovolemia is also present. Hypotension develops as cardiac output drops and tachycardia often ensues as a compensatory mechanism. Electrical alternans is a variation in the ECG caused by the shifting of the heart within the distended pericardium as it beats.

What of the following would decrease the amount of prolapse in patients with mitral valve prolapse? (select two) A. Increased ejection fraction B. Vasodilation C. Increased intravascular volume D. Hypertension

C. Increased intravascular volume D. Hypertension Any factor that maintains a larger ventricular volume will decrease the degree of prolapse. Hypertension, vasoconstriction, drug-induced myocardial depression, and increased preload will decrease the degree of prolapse.

Which agent is considered to be a balanced beta-1 and beta-2 receptor agonist? A. Epinephrine B. Ephedrine C. Isoproterenol D. Dopamine

C. Isoproterenol Isoproterenol is a balanced beta-1 and beta-2 receptor agonist with no alpha activity.

Dressler's syndrome is A. blindness related to hypotension in the prone position B. ST segment depression during myocardial ischemia C. pericarditis following a myocardial infarction D. low serum cortisol levels following chronic exogenous steroid administration

C. pericarditis following a myocardial infarction Dressler's syndrome is a form of pericarditis seen following myocardial infarction.

In a patient with mitral valve regurgitation, corrective surgery is preferred to be performed when the ejection fraction is A. 30% B. 40% C. 50% D. >60%

D. >60% Corrective surgery for mitral regurgitation should be done when the ejection fraction is greater than 60%. If the patient is symptomatic, it should be performed even if the ejection fraction is normal. Otherwise, significant cardiac remodeling occurs that prevents any effective improvement in performance.

Which of the following echocardiographic findings would be consistent with aortic stenosis? A. Eccentric hypertrophy of the right ventricle B. Concentric hypertrophy of the right ventricle C. Eccentric hypertrophy of the left ventricle D. Concentric hypertrophy of the left ventricle

D. Concentric hypertrophy of the left ventricle The exposure of the left ventricle to the increased pressure gradients associated with aortic stenosis result in concentric hypertrophy of the left ventricle (thickening of the ventricle wall). Eccentric hypertrophy is associated with the increased volume load associated with regurgitant lesions.

Which of the following is not a characteristic of hypertrophic cardiomyopathy? A. Occurs frequently in patients under the age of 30 B. Is associated with diastolic dysfunction C. Patients are typically asymptomatic at rest D. Is associated with a fixed aortic obstruction

D. Is associated with a fixed aortic obstruction Patients with aortic valve stenosis have a fixed obstruction, whereas about 25% of patients with hypertrophic cardiomyopathy exhibit a dynamic obstruction which peaks in mid-to-late systole, can vary from beat to beat, and can worsen with enhanced ventricular contractility, decreased ventricular volume, and decreased left ventricular afterload. Many patients are asymptomatic, but those that do exhibit symptoms exhibit dyspnea on exertion, fatigue, syncope, or angina. The first manifestation of hypertrophic cardiomyopathy in patients younger than 30 years is often sudden cardiac death, which is also the most common cause of death. Patients with hypertrophic cardiomyopathy typically suffer from diastolic dysfunction which is exhibited by increased left ventricular end-diastolic pressures despite an often hyperdynamic left ventricular function. The diastolic stiffness is due to the increased muscle mass of the left ventricle which is typically concentrated in the upper septum just below the aortic valve.

What induction agent is most useful for the patient with cardiac tamponade undergoing general anesthesia? A. Etomidate B. Propofol C. Thiopental D. Ketamine

D. Ketamine Induction is typically carried out with ketamine because it increases heart rate, contractility, and systemic vascular resistance. A benzodiazepine is often combined with it. The anesthetic may be maintained with nitrous oxide and fentanyl combined with pancuronium, which is useful for its vagolytic effects.

Which of the following congenital disorders would result in obstruction of right ventricular outflow? A. Aortic stenosis B. Mitral stenosis C. Tricuspid stenosis D. Pulmonic stenosis

D. Pulmonic stenosis Pulmonic stenosis produces obstruction to right ventricular outflow.


Kaugnay na mga set ng pag-aaral

Foundations Exam 1 Practice Test

View Set

Exam 1- PEDS fluid and electrolytes

View Set

Patient Care- Chapter 18 Aseptic Techniques

View Set

Chapter 12 - Cash Flow Estimation

View Set

Interpersonal Communication Chapter 11 (Romantic Relationships)

View Set

Physics Exam 2 (Multiple Choice)

View Set

Chapter 1: Economics & Analysis - Fundamental Analysis

View Set

Vocabulary Workshop Second Course - Lesson 26

View Set